Tehát $ {\ bf R} ^ {n \ alkalommal p} $ értékben megvan a Frobenius belső termék adta: $$ \ langle A, B \ rangle = \ text {tr} (A ^ TB) $$

amely euklideszi belső termékként értelmezhető a $ {\ bf R} ^ {np-n } $. Megértésem szerint a $ {\ bf R} ^ {np} $ összes belső terméke $$ a ^ TPb $$ néven írható $ P $ pozitív-határozott értékre. A legjobb, amit a Frobenius belső termékének $ {\ bf R} ^ {n \ times p} $ -on való kiterjesztésére tettem, ami a $$ \ langle A, B \ rangle = \ sum_ {i = 1} formájú. ^ N \ text {tr} ((X_iAY_i) ^ T (X_iBY_i)) $ $ for $ X_i \ in {\ bf R} ^ {m_i \ times n} $ és $ Y_i \ in {\ bf R} ^ {p \ szor q_i} $ teljes rangot. Szeretném azonban tudni, hogy ez magában foglalja-e a $ {\ bf R} ^ {np} $ összes belső termékét, vagy esetleg az elbocsátások miatt a szükségesnél bonyolultabb.

Megtalálhatom a megfelelő mátrix belső termék megfelelő $ P $ mátrixa, ha a $ {\ bf R} ^ {n \ nszeres p} $ szabványos alapját veszi és a mátrixot alkotja

\ begin {bmatrix} \ langle E_1 , E_1 \ rangle & \ langle E_1, E_2 \ rangle & \ dots & \ langle E_1, E_ {np} \ rangle \\ \ langle E_2, E_1 \ rangle & \ langle E_2, E_2 \ rangle & & \ vdots \\ \ vdots & & \ ddots \\ \ langle E_ {np }, E_1 \ rangle & \ dots & \ dots & \ langle E_ {np }, E_ {np} \ rangle \ end {bmatrix}

de nem tudom, hogy a fenti mátrix belső szorzat általános formája az összes pozitív-meghatározott mátrixot lefedi-e $ P $.

Frissítés:

a kérdés újabb verziója itt: MathOverflow: https://mathoverflow.net/questions/229675/extending-the-trace-inner-product-to-all-matrix-real-inner-products

Megjegyzések

  • Üdvözöljük a SciComp.SE oldalon! Ez egy érdekes kérdés, de sokkal megfelelőbbnek tűnik a math.stackexchange.com számára. (Hacsak ' nem kapcsolódik egy számítástechnikai problémához, I ' m hiányzik, ebben az esetben ' legyen jó, ha ezt hozzáadhatnád.)
  • @ChristianClason, ez ' s a mátrix sokaságokon történő optimalizáláshoz kapcsolódik Riemannian metrikákkal, mivel Riemannian a metrikák az érintőtér belső termékei. ' szinte biztos, hogy túl fejlett a Math.SE számára, az egyetlen megfelelő hely a MathOverflow lenne. Valójában megtalálhattam azt a véleményem szerint megoldást, amelyet válaszként közzétehetek, miután elvégzem a rendetlenséget, és bizonyítom, hogy ez megoldás, de ha ' át szeretne költözni ezt a MathOverflow-hoz I ' m ok ezzel. <

Hozzáadom az optimalizálási kontextust, amikor esélyem lesz rá.

  • A $ P $ mátrixnak szimmetrikusnak is kell lennie, nem csak pozitívnak.
  • @WolfgangBangerth, a pozitív-határozott szimmetrikus.
  • Nem minden szerző esetében a pozitív-határozottság szimmetriát jelent.
  • Válasz

    A belső termék egy műveletnek tekinthető $ f (a, b) = \ left < a, b \ right > $, azaz egy bilináris függvény, amely (i) nem negatív számot ad vissza, (ii) kielégíti az $ f (a, b) = f (b, a) $.

    Az $ a, b \ vektorokban a mathbb R ^ n $ vektorokban minden bilináris függvény, amely kielégíti ezeket a tulajdonságokat, $$ f (a, b) = \ sum_ {i, j = 1 } ^ n a_i P_ {ij} b_j $$ ahol $ P $ szimmetrikus és pozitív határozott. Az $ a, b \ in \ mathbb R ^ {n \ szorzat p} $ mátrixok esetében az összes ilyen függvény felírható $$ f (a, b) = \ sum_ {i, k = 1} ^ n \ sum_ { j, l = 1} ^ p a_ {ij} P_ {ijkl} b_ {kl} $$, ahol most $ P $ egy 4. rangú tenzor, amely szimmetrikus abban az értelemben, hogy $ P_ {ijkl} = P_ {klij} $ és pozitív határozott abban az értelemben, hogy $ f (a, a) > 0 $ minden $ a \ neq 0 $ esetén.

    A kérdésed hogy minden ilyen feltételnek megfelelő $ P $ megírható-e egy formát, amely a $ X_i, Y_i $ vektorokból származik Úgy gondolom, hogy erre nemleges a válasz. Ez egyszerűen azért van így, mert (az egyszerűség kedvéért feltételezve, hogy $ n = p $) a szimmetrikus $ P $ (aszimptotikusan) $ n ^ 4/2 $ szabadságfokkal rendelkezik, míg a $ n $ vektorok $ X_i, Y_i $ csak $ 2n ^ 2 $ szabadságfok. Más szavakkal, nem hiszem, hogy kellően nagy $ n $ esetén a megközelítésednek elég sok szabadságfoka van.

    Megjegyzések

    • I valójában azt hiszem, hogy a válasz igen, én ' ezt a kérdést a matematika túlcsordulásával újra feltöltöm a frissített eredményeimmel.
    • Igen, az az érv, hogy a # paraméter növekszik kvartikusan a vektor belső termékterében, míg a mátrixban csak kvadratikusan a belső terméktér kényszerítő, azonban mivel a tér véges véges, ezt képesnek kell lennünk legyőzni a $ N $ megfelelő növelésével.
    • Elnézését kérem, a kérdés újabb verzióját tettem közzé a MathOverflow-n, azonban ' megfelelően frissítve gondoltam, hogy megfelelő, itt van a link arra az esetre, ha szeretné hogy átküldje a válaszát, vagy frissítse a választ az újabb verzió alapján. mathoverflow.net/questions/229675/…
    • @Thoth Ne feledje, hogy @ ChristianClason tanácsot adott tegye fel kérdését a math.stackexchange.com webhelyre, ne a mathoverflow.net webhelyre. Ez két különböző webhely, különböző célokkal és közönséggel.
    • @FedericoPoloni igen, tudom, és ha elolvassa, amit írtam, azt mondtam neki, hogy azt gondoltam, hogy túl fejlett a Math.SE számára, és nem valószínű, hogy megkapja választ ott.

    Vélemény, hozzászólás?

    Az email címet nem tesszük közzé. A kötelező mezőket * karakterrel jelöltük